LSAT and Law School Admissions Forum

Get expert LSAT preparation and law school admissions advice from PowerScore Test Preparation.

 Administrator
PowerScore Staff
  • PowerScore Staff
  • Posts: 8916
  • Joined: Feb 02, 2011
|
#43369
Please post your questions below! Thank you!
 chian9010
  • Posts: 81
  • Joined: Jun 08, 2018
|
#57220
I tried to come up with my own answers before I checked the answer choices for this question.

From the stimulus, the legistor's conclusion is the recently passed highway bill is clearly very unpopular with voters.
And he got this conclusion from seeing the polls result which shows that majority party, which supported the bill's passage, will lose more than a dozen seats in the upcoming election.

I think there are two flows
The first one is that it seems to me that the majority part being unpopular among voters might be just a correlation that majority of them supported the bills. We cant determine the causal effect from the simple correlation relationship. Therefore, we can't say those party being unpopular by voters because they supported the bills. There might be other factors that cause the majority part that are not popular among voters.

The second is the poll result. If passed highway bill is truly unpopular with "certain group" of voters, it is highly possible that those certain group of voters will be more likely to participate the polls than those who don't care about passing of the bills.

So when I read the answer choice I was trying to find the answer that mentions about correlation or poll result inaccuracy. I chose C because it seems to me that the legislator presuppose the causal relationship that connect majority unpopular to bill unpopular.

Could you explain why A is correct?
 student987
  • Posts: 28
  • Joined: Apr 09, 2018
|
#57666
Hello! I am also really confused about what exactly the flaw here is. Could anyone clarify this question? Thank you!
 Malila Robinson
PowerScore Staff
  • PowerScore Staff
  • Posts: 296
  • Joined: Feb 01, 2018
|
#58001
Hi Student 987 Chian9010,

This is an evidence error related to the conclusion.

The conclusion states: "The recently passed highway bill is clearly very unpopular with voters." So then we have to ask what makes it so clear (what is the evidence) that the voters do not like the bill?

But the only evidence that is provided is that polls have said that the party supporting the bill are going to lose seats.

So we are left to ask things like: Were the polls conducted before or after the bill was proposed? Were there other reasons, such as scandals, that would have caused the party to lose seats? Were the polls conducted in an unbiased way? (and more)

We do not have the answers to those types of questions, so the polls that are mentioned only tell us about voters thoughts about the party, they really don't tell us anything about their feelings towards the highway bill. That makes it possible that with or without the bill, voters may have had the same poll results. That leads to (A) as the correct answer.

Hope that helps!
-Malila
 LSAT2018
  • Posts: 242
  • Joined: Jan 10, 2018
|
#58783
I thought the answer was (C) because the premise provides that polls predict that they will lose seats in the election because they supported the passage. Doesn't this imply that the author presumes the bill is unpopular? Or is this an additional assumption that is not indicated in the answer, which makes it incorrect?
 Pragmatism
  • Posts: 68
  • Joined: Jan 11, 2018
|
#63261
I had a really hard time with this question. This was my thought process:
—How is the conclusion—the recently passed bill is clearly very unpopular with voters— supported by the premise—polls predict that the majority party, which supported the bill, will lose seats in the upcoming election?
—I also thought, while there is no timeline associated with the timing of the polls and the passage of the bill, I assumed the legislator was subtly attacking the majority party to drive home his point.
—So, since nothing came close to such an answer choice, I went with C :x

Is A saying that the legislator was assuming that it had to do something with the majority party and thus, attacking them in doing so?
 Brook Miscoski
PowerScore Staff
  • PowerScore Staff
  • Posts: 418
  • Joined: Sep 13, 2018
|
#63320
LSAT2018,

The trick to this question is to identify the error in reasoning, which is causal. The stimulus assumes that the polls have a causal relationship to the support for the bill; instead, the majority party could be losing votes for some other reason. That is why the answer is (A)--the answer that separates the cause from the effect. Alternatively, you can see this as an interpretation of evidence issue. The party is losing seats, and the party supports the bill. The stimulus interprets that there is a link, and (A) states that there is not a link.

The reason that (C) is wrong is that (C) is standard language used to describe circular reasoning. Here, there is evidence--the party is losing seats, and the party supports a bill. The reasoning might not be stunning, but it does involve an attempt to interpret evidence. On the LSAT, you should be wary of a choice that describes circular reasoning or tautology when there is an attempt to interpret evidence, although it is still a possible choice.

If I were to modify this stimulus to create circular reasoning, it would look something like this:

I know that the recently passed bill is very unpopular with voters. After all, polls predict that the majority party that supported the bill will loose seats. Other analysts say that voters were neutral about the bill and opposed the majority party for another reason, but they are wrong because the bill is very unpopular with voters.

That pattern has shown up on the LSAT, and if you see that pattern, either (A) or (C) could work [but of course you would only be given one of the two choices to pick].
 Brook Miscoski
PowerScore Staff
  • PowerScore Staff
  • Posts: 418
  • Joined: Sep 13, 2018
|
#63321
Pragmatism,

In this case, paying attention to leading/transition words will help you interpret the stimulus correctly. Here, the legislator used the phrase "after all," which means that he believes that the second sentence supports the first sentence. Understanding that, you have his argument--we know the bill is unpopular because the voters are kicking out the party that supports it. Looking at (A), if the party was going to loose anyway, its support for the bill doesn't tell us much, so (A) works. Read over the response to LSAT2018 with regard to (C). In general, you should not pick an answer choice describing circular reasoning when there has been an attempt to interpret evidence, even if the interpretation is pretty shoddy.
User avatar
 fortunateking
  • Posts: 31
  • Joined: Jan 10, 2022
|
#100654
In the stimulus the author use AFTER ALL which indicates the premise, and figuring out the reasoning structure is the very first step to attack any LR question:
Premise: The party supports the bill and is losing seats
Conclusion: The bill is unpopular with voters
To make the argument work, the author has made an unwarranted assumption, that supporting the bill is at least partly the reason why the party is losing seats.
(A) is correct by pointing out that supporting the bill may not be a reason. (the party is losing anyway)
 Rachael Wilkenfeld
PowerScore Staff
  • PowerScore Staff
  • Posts: 1358
  • Joined: Dec 15, 2011
|
#100704
Spot on, fortunateking. "After all" is one of those tricky premise indicators because it can sound conclusory. But it's always indicating a reason to believe something, and that's going to be a premise. Great work!

Get the most out of your LSAT Prep Plus subscription.

Analyze and track your performance with our Testing and Analytics Package.